The Hart-Scott-Rodino Act requires certain competing business entities who are contemplating mergers involving dollar amounts of a certain size to give advance notice to the: I. Federal Trade Commission. II. Federal Antitrust Bureau. III. Department of Justice. IV. Secretary of Commerce. Multiple Choice III only. III and IV. I only.

Answers

Answer 1

Answer:

I. Federal Trade Commission.

III. Department of Justice.

Explanation:

The Hart-Scott-Rodino Act of 1976 is actually a complement of prior antitrust laws that already existed but were incomplete or outdated. Its main focus is set on a pre-merger notification report that must be given by the parties involved to the Federal Trade Commission (FTC) and the Department of Justice (DOJ). The HSR Act only applies to mergers or acquisitions that are relatively large, e.g. the transaction must involve at least $90 million.


Related Questions

Locher Co. reported cost of goods sold for 2020 of $1,400,000. Locher later discovered that its ending inventories at December 31, 2019 and 2020 were overstated by $110,000 ad $35,000, respectively. What is the correct amount for 2020 cost of goods sold

Answers

Answer:

$1,325,000

Explanation:

Calculation for the correct amount for 2020 cost of goods sold

Using this formula

2020 Correct amount for Cost of Goods sold = Reported cost of Goods sold + 2020 Overstated ending inventory - 2019!Overstated Ending inventory

Let plug the formula

2020 Correct amount for Cost of Goods sold = $1,400,000 + $35,000 - $110,000

2020 Correct amount for Cost of Goods sold = $1,325,000

Therefore 2020 Correct amount for Cost of Goods sold will be $1,325,000

Juanita Domingo's parents want to establish a college trust for her. They want to make 16 quarterly withdrawals of $2000, with the first withdrawal 3 months from now. If money is worth 7.5%, compounded quarterly, how much must be deposited now to provide for this trust

Answers

Answer:

Initial investment= 26,921.47

Explanation:

Giving the following information:

Withdrawls= 16 of $2,000

Interest rate= 0.075/4= 0.01875

To calculate the initial investment, first, we need to calculate the future value of the investment:

FV= {A*[(1+i)^n-1]}/i

A= quarterly payment

FV= {2,000*[(1.01875^16) - 1]} / 0.01875

FV= $36,918.86

Now, the present value:

PV= FV/(1+i)^n

PV= 36,918.86/1.01875^17

PV= $26,921.47

Initial investment= 26,921.47

By making many burgers before customers ask for them, a fast-food restaurant is sacrificing the dimension of ______ and improving the dimension of ______ in the customers' utility function.

a. fit, timing
b. fit, price
c. fit, timing
d. timing, fit

Answers

Answer:

a. fit, timing

Explanation:

A utility function can be defined as a measure of the benefits or satisfaction derived by the consumer of finished goods such as snacks, perfume, shoes, clothes, electronic gadgets etc.

Basically, a customer’s utility function typically refers to the welfare, enjoyment, benefit, usefulness or satisfaction a particular customer derives from a product or goods.

The dimension of timing and fit plays a very significant role in the process of adding value, benefits or satisfaction to a customer's experience.

By making many burgers before customers ask for them, a fast-food restaurant is sacrificing the dimension of fit and improving the dimension of timing in the customers' utility function.

This year, Barney and Betty sold their home (sales price $750,000; cost $200,000). All closing costs were paid by the buyer. Barney and Betty owned and lived in their home for 18 months. Assuming no unusual or hardship circumstances apply, how much of the gain is included in gross income

Answers

Answer: $550,000

Explanation:

From the question, we are informed that Barney and Betty sold their home (sales price $750,000; cost $200,000) and that all the closing costs were paid by the buyer.

Since no unusual or hardship circumstances apply and all the closing stocks were paid by the buyer, the amount of the gain that will be included in gross income will be:

= $750,000 - $200,000

= $550,000

Y3K, Inc., has sales of $12,840, total assets of $4,730, and a debt-equity ratio of .25. If its return on equity is 14 percent, what is its net income

Answers

Answer:

Equity multiplier = 1 + Debt−equity ratio

Equity multiplier = 1 + 0.25

Equity multiplier = 1.25

Total asset turnover = Sales / Total assets

Total asset turnover = $12,840 / $4,730

Total asset turnover = 2.71 times

ROE = (Profit margin)(Total asset turnover)(Equity multiplier)

0.14 = (Profit margin)(2.71)(1.25)

Profit margin = 0.0413 or 4.13%

Profit margin = Net income / Sales

0.0413 = Net income / $12,840

Net income = $529.76

 

The Sour Pickle Company has a debt-equity ratio of 1.37. Return on assets is 7.62 percent,...

The Sour Pickle Company has a debt-equity ratio of 1.37. Return on assets is 7.62 percent, and total equity is $685,000. a. What is the equity multiplier? (Do not round intermediate calculations and round your answer to 2 decimal places, e.g., 32.16.) b. What is the return on equity? (Do not round intermediate calculations and enter your answer as a percent rounded to 2 decimal places, e.g., 32.16.) c. What is the net income? (Do not round intermediate calculations and...

Shelton Company has a debt-equity ratio of 1.41. Return on assets is 7.66 percent, and total...

When computing economic growth, changes in nominal gross domestic product (GDP) must be adjusted to reflect population growth because:

Answers

Answer:

if real GDP remains the same, an increase in the population actually means a lower average standard of living

Explanation:

Economic Growth is a increase in productive capabilities of an economy over a given period of time, usually a year. the economy is able to satisfy more of it's consumers wants and need.

GDP is said to be is total value of all finale goods and services produced within an economy in a given period of time, usually a year.nominal or price GDP is a GDP with no adjustment for price.

Ben & Jerry's mission to make the world a better place is linked to various organizational and marketing strategies, one of which is:_______a. supporting farmers who agree to use sustainable farming practices, implementing fair working standards, and investing in local communities.b. the intent of making modest profits without sacrificing high product quality standards.c. ingredients that are all completely organic and are available only in Vermont to ensure freshness and contribute to the local economy.d. a commitment to donating a percentage of profits to Teach for America.e. ingredients that come exclusively from developed countries promoting Fair Trade practices.

Answers

Answer:

a. supporting farmers who agree to use sustainable farming practices, implementing fair working standards, and investing in local communities.

Explanation:

Ben and Jerry refers to an agricultural company that is focused on delivering the best of its finished goods to the consumer.

Ben & Jerry's mission to make the world a better place is linked to various organizational and marketing strategies, one of which is supporting farmers who agree to use sustainable farming practices, implementing fair working standards, and investing in local communities.

Bob bought some land costing $15,690. Today, that same land is valued at $45,417. How long has Bob owned this land if the price of land has been increasing at 5 percent per year

Answers

Answer:

t = 21.78436854 rounded off to 21.78 years

Explanation:

We are given the future value and the present value of land. To calculate the number of years of ownership of the land whose price has been increasing at 5% per year, we can use either use the formula for Future Value or Present value.

Here we are solving it using the future value formula which is,

FV = PV * (1 + r)^t

Where,

FV is Future Value

PV is Present value

r is the annual rate of increase

t is time period in years

Plugging in the values for FV, PV and r, we can calculate the value of t,

45417 = 15690 * (1 + 0.05)^t

45417 / 15690 = (1+0.05)^t

2.894646272  =  (1.05)^t

Taking log on both sides.

Ln(2.894646272) / Ln(1.05)  =  t

t = 21.78436854 rounded off to 21.78 years

Firm X is considering the replacement of an old machine with one that has a purchase price of $90,000. The current market value of the old machine is $24,000 but the book value is $34,000. The firm's combined tax rate is 26%. What is the net cash outflow for the new machine after considering the sale of the old machine

Answers

Answer:

Net cash outflow for new machine = $63,400

Explanation:

The computation of the net cash outflow for the new machine is shown below:

Sale proceeds from old machine = $24,000

Add  Tax Savings occured from loss on sale ($34,000 - $24,000) ×  0.26 = $2,600

Net benefit arise from the sale of old machine = $26,600

Less:

Cost of new machine = $90,000

So,

Net cash outflow for new machine = $63,400

An online buying club requires an annual fee of $130 in order to qualify for a 16 percent discount on purchases. What amount would a person have to buy in order to save enough to cover the cost of the membership

Answers

Answer:

$812.50

Explanation:

Calculation for the amount a person have to buy in order to save enough to cover the cost of the membership

Using this formula

Amount to buy =Annual fee/Discount on purchases

Let plug in the formula

Amount to buy=$130/0.16

Amount to buy=$812.50

Therefore the amount a person have to buy in order to save enough to cover the cost of the membership is $812.50

When activities are arranged in a line according to the sequence of operations for assembling a particular product the layout is said to be a:

Answers

Answer: Product layout

Explanation:

Product layout is when activities are arranged in a line according to the sequence of operations for assembling a particular product.

In product layout, both the tools and th supplies which are needed for the manufacturing process are at the assembly line. This brings about identical products to be made.

Like all entrepreneurs, Jennifer Hyman and Jennifer Fleiss recognized an opportunity they could exploit through technology, but they needed resources, such as capital and knowledge, to enter the market. How did they acquire these resources?

Answers

Answer: By securing funding from investors and hiring people with complementary skills

Explanation:

Entrepreneurs can acquire capital from other people who will then become investors and and give them funding. With this funding they will initiate the business they had in mind and provide returns for the investors.

Entrepreneurs do not always have to have all the skills required to run their business because they can simply employ people who do. These people with complimentary skills will ensure that the company has the knowledge required to do well.

A car may be purchased with $3,500 down payment now and 72 monthly payments of $489. If the interest rate is 9% per year compounded monthly, what is the price of the car

Answers

Answer: $30628.155

Explanation:

After the calculations in the attachment has been made, the present value was gotten as $27128.55

The price of the car was calculated as the addition of the down payment and the present value which was:

= $3500 + $27128.55

= $30628.155

Note that in the calculation in the attachment,

r = 9%/12 = 0.75%

n = 72.

Check attachment for more details

What are 3 primary ways to get information into QuickBooks Online?

Answers

Answer:

• Use the import data tool.

• Manually place data in QuickBooks online formats and check registers.

• Convert data from existing QuickBooks table file

Explanation:

QuickBooks is simply an accounting software package. It should be noted that QuickBooks products offer accounting applications that are used to make payments , bills and also perform payroll functions.

The three primary ways to get information into Quickbooks Online are the use the import data tool, manually placing data in QuickBooks online formats and check registers and converting data from existing QuickBooks table file.

The Roscoe Company's March 31 bank statement balance was $79,500. As of March 31, outstanding checks total $25,800 and deposits in transit total $16,900. Assuming there are no other reconciling items, what was the March 31 cash balance on Roscoe's books?
a. $70,600
b. $79,500
c. $122,200
d. $88,400

Answers

Answer:

a. $70,600

Explanation:

Calculation for the cash balance on Roscoe's books

Using this formula

Cash balance = Bank balance - Outstanding checks + Deposits in transit

Let plug in the formula

Cash balance = $79,500 -$25,800 + $16,900

Cash balance= $70,600

Therefore the cash balance on Roscoe's books will be $70,600

Country X has requested and received a loan from the world bank. Country X may not use the money to finance ____

Answers

Answer:

Non-developmental Projects

Explanation:

Country X requested for and received a loan from the world bank (formerly the International Bank for Reconstruction and Development).

Country X may not (Country X is not allowed to) use the money to finance projects that are unrelated to development.

This stems from the purpose of the World Bank and its funding acts. The World Bank's key function is:

- Lending to the middle-class nations (at relatively low interest rates) and giving to the poorest nations (at no interest rate) who are its members.

Country X hence is either a middle-income country or a low-income country. It may only use the loan to finance projects that aim at improving the quality of human life e.g. health projects, educational projects, environmental projects, etcetera.

A construction manager just starting in private practice needs a van to carry crew and equipment. She can lease a used van for $3,576 per year, paid at the beginning of each year, in which case maintenance is provied. Alternatively, she can buy a used van for $6,461 and pay for maintenance herself. She expects to keep the van for three years at which time she could sell it for $1,437. What is the most she should pay for uniform annual maintenance to make it worthwhile to buy the van instead of leasing it, if her MARR is 20%

Answers

Answer:

The answer is "1832".

Explanation:

The PVAF(r,n) start value[tex]= \frac{(1+r)^{n-1} -1 \ \ +1}{r \times (1+r)^n-1}[/tex]

The PVAF(r,n) end value[tex]= \frac{(1+r)^{n} -1}{r \times (1+r)^n}[/tex]

The PVF(r,n)  value [tex]= \frac{1}{(1+r)^n}[/tex]

[tex]r = \text{Rate of dirscount} \\\\n = terms[/tex]

[tex]\text{Present Value= Monthly rent} \times PVAF[/tex]

PVAF(20%, 3) starting = 2.5278

PVAF(20%, 3) ending = 2.1065

PVAF(20%, 3) starting = 5787

calculating the lease payment of present value:

[tex]present \ value = Lease\ payment \times PVAF(10 \%,3) starting[/tex]

                      [tex]=3609 \times 2.5278\\\\=9122.75[/tex]

existing importance by busing excluding annual servicing

[tex]\text{present value = cost - sale value} \times PVF(20 \%,3)[/tex]

                     [tex]=6113-1469\times 0.5787 \\\\=5262.88[/tex]

Calculating the annual maintanance=[tex]\frac{\text{present value of leasing -buying}}{ PVAF(20 \%,3) ending}[/tex]

                                                            [tex]=\frac{3859.87}{2.1065}\\\\=1832.32 \ \ \ or \ \ \ 1832[/tex]

3.Assuming the quotes of exchange rates against dollar for the Swiss franc and the Australian dollar are as follows: Bank QuotationsBidAsk $ / SFr .7265 .7272 $ / A$ .6135 .6140 Suppose a Swiss company has investment proceeds equal to 1 million Australian dollars, and wants to convert it to Swiss franc. What are his proceeds from conversion

Answers

Answer:

proceeds from conversion is SFr 843,646.86

Explanation:

The computation of the proceeds from the conversion is shown below:

By Converting A$ into $ so we get

=  1,000,000 × 0.6135

= $613,500

And, if

 Convert $ into Sfr

So,

= $613,500 ÷ 0.7272

= Sfr 843,646.8646

Therefore, proceeds from the conversion is SFr 843,646.86

The same is to be considered so that the correct value could come

Every employee at a fitness center is required to wear a crisp white uniform. Towels are neatly rolled and stacked, the floors glisten with cleanliness, and there is always a beautiful display of fruit in the hallway. Based on this information, the fitness center prioritizes which variable of service quality

Answers

Answer:

Tangibles.

Explanation:

Tangibles in this question is directly the variable of service quality exuded. This is because it refers to physical facilities, equipment, and appearance of a service firm’s employees. the task of the tangible and physical evidence of a service is multifunctional. When a patient within the lounge of a clinic sees the doctor’s certificate, he becomes attentive to the standard of service he's getting ready to receive. Example is seen when a dental clinic provides patients with clean rubber footwear and freshly laundered bibs or coats before the particular service, the patients and their accompanying relatives or friends are going to be impressed. A dentist wearing a spotless white coat is likely-to impress, them even further. Tangibles provide the customer proof of the standard of service.

Apricot Corporation distributes property ($125,000 basis and $150,000 fair market value) to its sole shareholder, Ellie. The property is subject to a liability of $200,000, which Ellie assumes. Apricot has E

Answers

Answer: $75,000; $400,000.

Explanation:

Here is the complete question:

Apricot Corporation distributes property ($125,000 basis and $150,000 fair market value) to its sole shareholder, Ellie. The property is subject to a liability of $200,000, which Ellie assumes. Apricot has E & P of $325,000 prior to the distribution. a)What gain, if any, does Apricot recognize on the distribution? What is Apricot’s accumulated E & P at the start of the following year?

The gain recognized by Apricot on the distribution will be:

= $200,000 - $125,000

= $75,000

Apricot’s accumulated E & P at the start of the following year will be:

= $325,000 + $75,000

= $400,000

KLM Corporation's quick assets are $6,158,000, its current assets are $13,650,000 and its current liabilities are $8,170,000. Its acid-test ratio equals:

Answers

Answer:

0.75

Explanation:

KLM corporation quick assets is $6,158,000

Current assets are $13,650,000

Current liabilities are $8,170,000

Therefore the acid test ratio can be calculated as follows

= quick assets/current liabilities

= $6,158,000/$8,170,000

= 0.75

Hence the acid test ratio is 0.75

how old does a fight have to be in order for it to be old

Answers

Answer:

2 weeks and more why?

Explanation:

Assume that Shavonne's marginal tax rate is 37 percent and her tax rate on dividends is 20 percent. If a corporate bond pays 8.8 percent interest, what dividend yield would a dividend-paying stock (with no growth potential) have to offer for Shavonne to be indifferent between the two investments from a cash-flow perspective?

Answers

Answer:

Interests received from corporate bonds is taxed as ordinary income = 8.8% x (1 - 37%) = 5.544% after tax yield

Non-qualified dividends are also taxed as ordinary income, therefore, the after tax yield of a non-qualified dividend that yields 8.8% = 5.544%

Qualified dividend pay a lower tax rate, since they are taxed as long term capital gains. If the after tax yield will be the same, then:

5.544% = dividend yield x (1 - 20%)

5.544% / 0.8 = dividend yield

6.93% = dividend yield

A firm has a capital structure with $100 million in equity and $100 million of debt. The cost of equity capital is 15% and the pretax cost of debt is 7%. If the marginal tax rate of the firm is 25%, compute the weighted average cost of capital of the firm. Group of answer choices

Answers

Answer:

The weighted average cost of capital of the firm is 10.125%

Explanation:

The computation of the weighted average cost of capital is shown below:

= Cost of equity × weight of equity + cost of debt × (1 - tax rate) × weight of debt

= 15% × $100 ÷ ($100 + $100) + 7% × (1 - 0.25) × $100 ÷ ($100 + $100)

= 15% × 0.5 + 7% × 0.75 × 0.5

= 7.5% + 2.625%

= 10.125%

Hence, the weighted average cost of capital of the firm is 10.125%

We simply applied the above formula

If you co-sign for a friend's credit card, what is the danger to you if your friend fails to pay?
A.
You might get secured credit.
B.
You might have to pay extra money in taxes.
C.
Your credit score might go down.
D.
Your credit score might go up.


Please select the best answer from the choices provided

A
B
C
D

Answers

Answer:

C

Explanation:

You want to have $60,000 in your savings account 12 years from now, and you’re prepared to make equal annual deposits into the account at the end of each year. If the account pays 6.4 percent interest, what amount must you deposit each year?

Answers

Answer:

$3,474.39

Explanation:

The amount that you need to deposit each year (PMT) is calculated as follows:

FV = $60,000

N = 12

P/yr = 1

i = 6.4 %

PV = $0

PMT = ?

Using a Financial Calculator to input the values as shown, the deposit each year would be $3,474.39

to determine the regular payment​ amount, rounded to the nearest dollar. In terms of paying less in​ interest, which is more economical for a ​$ ​mortgage: a​ 30-year fixed-rate at ​% or a​ 20-year fixed-rate at ​%? How much is saved in​ interest?

Answers

Answer:

the numbers are missing, so I looked fro similar questions:

Which is more economical for a $210.000 mortgage: a 30-year fixed-rate at 9% or a 20-year fixed-rate at 8.5%? How much is saved in interest?

if you take the 30 year mortgage, your monthly payment will be:

monthly payment = present value / PV annuity factor

present value = $210,000PV annuity factor, 0.75%, 360 periods = 124.2817

monthly payment = $210,000 / = $1,689.71

total payments = $1,689.71 x 360 = $608,295.60

total interests paid = $608,295.60 - $210,000 = $398,295.60

if you take the 20 year mortgage, your monthly payment will be:

monthly payment = present value / PV annuity factor

present value = $210,000PV annuity factor, 0.7083%, 240 periods = 115.2308

monthly payment = $210,000 / = $1,822.43

total payments = $1,822.43 x 240 = $437,383.20

total interests paid = $437,383.20 - $210,000 = $227,383.20

if you take the 20 year mortgage, you will save $398,295.60 - $227,383.20 = $161,912.40 in interests

When sales units go up by 25% (assuming the unit selling price and unit variable expense are constant): A. Variable expenses go up by 25%. B. Net income will go up by 25%. C. Fixed expenses will go up by 75%. D. Contribution margin will go up by 75%.

Answers

Answer: A. Variable expenses go up by 25%.

Explanation:

Total variable expenses are calculated on the basis of the entire units to be sold which means that if sales units were to increase by 25%, the total variable expenses will increase by 25% as well.

For instance, If selling price per unit is $4 and variables, $3 and the number of units to be sold is 20, sales will be $80 and variable costs will be $60.

Assuming units went up by 25% to 25 units, variable costs would be; $75 which is an increase of;

= (75 - 60) / 60

= 25%

Paid $78,000 cash to replace a motor on equipment that extends its useful life by four years. Paid $390 cash per truck for the cost of their annual tune-ups. Paid $312 for the monthly cost of replacement filters on an air-conditioning system. Completed an addition to a building for $438,750 cash. 1. Classify the above transactions as either a revenue expenditure or a capital expenditure. 2. Prepare the journal entries to record transactions a and d.

Answers

Answer: a. Capital expenditure

b. Revenue expenditure

c. Revenue expenditure

d. Capital expenditure

Explanation:

Capital expenditures are usually huge expenditure on fixed assets such as land or building and they re usually incurred to generate revenue for the business.

Revenue expenditures are usually for short term basis and are operating expenses, that us required to run the business daily.

Based on the above explanation, the answers to the following will be:

a. Paid $78,000 cash to replace a motor on equipment that extends its useful life by four years. - Capital expenditure

b. Paid $390 cash per truck for the cost of their annual tune-ups. - Revenue expenditure

c. Paid $312 for the monthly cost of replacement filters on an air-conditioning system. - Revenue expenditure.

d. Completed an addition to a building for $438,750 cash. - Capital expenditure

Check the attachment for the journal entry

A large electric utility company spews million tons of greenhouse gases​ (mostly carbon​ dioxide) into the environment each year. This company has committed to spending ​$ billion in capital over the next five years to reduce its annual emissions by ​%. More will be spent after five years to reduce greenhouse gases further. a. What is the implicit cost of a ton of greenhouse​ gas? b. If the United States produces billion tons of greenhouse gases each​ year, how much capital must be spent to reduce total emissions by ​% over the next five years based on your answer in Part​ (a)?

Answers

Answer:

a. $537.31 per ton

b. $64,477,200,000

Explanation:

a. 5% of 67 million tons will cost $1.8 billion to be reduced.

The implicit cost of a ton therefore is;

= 1,800,000,000 / (5% * 67,000,000)

= 1,800,000,000/ 3,350,000

= $537.31 per ton

b. The amount to be spent to reduce 4 billion tons by 3% is;

= 537.31 * (4,000,000,000 * 3%)

= $64,477,200,000

Other Questions
solve for x 4x-12= 2(2x-6) Which of the following are typical responses to an authors or speakers use of pathos?Select all that apply.sympathylaughtertrustanalysisdisgust Find the indicated probability. The age distribution of students at a community college is given below. Age (years) Number of students (f) 1109 A student from the community college is selected at random. Find the probability that the student is at least 31. Round your answer to three decimal places. A jacket usually sells for $62.00. If the jacket is 20% off, and sales tax is 5%, what is the total price of the jacket, including tax? Managerial accounting information: Multiple Choice Is used mainly by external users. Involves gathering information about costs for planning and control decisions. Is generally the only accounting information available to managers. Can be used for control purposes but not for planning purposes. Has little to do with controlling costs. ? QuestionA third-degree polynomial function has three real zeros, has a relative minimum at (2,-3), and increases on the interval-infinity < x < -3. Sketch the graph of a function with these features. What did the Stamp Act tax?the anwsers down below which one is bestA.Paper goodsB.Postage stampsC.Homes 3x+2x+4=39 multi steps equation Solve the following system of equations: 5x + y = 9 3x + 2y = 4 A (2, 5) B (1, 4) C (2, 1) D (4, 4) plss Spanish speaker help me Effect of Doubtful Accounts on Net IncomeDuring its first year of operations, Mack's Plumbing Supply Co. had sales of $630,000, wrote off $10,100 of accounts as uncollectible using the direct write-off method, and reported net income of $69,300. Determine what the net income would have been if the allowance method had been used, and the company estimated that 2.5% of sales would be uncollectible. Ryan wants to plant 1/5 packet of seeds in each row of his garden he has four packets of C Ryan uses the expression 1/5 / 4 to find the number of rows you can plant explain what is wrong with his expression then write a equation to show the correct number of rows Based on this vessel, what can we say about the influence that Teotihuacan had on classic Maya society? What happened as the number of public schools in the South began to grow? A. Blacks got the same education as whites. B. Both whites and blacks went to the same schools. C. Blacks and whites were kept in separate schools. D. Blacks asked to have separate schools. Ernesto scored twice as many goals at his last field hockey game as he did during the first game of the season. If g represents the number of goals Ernesto scored during the first game, which expression represents the number of goals he scored during the last game? Part AIn "Close Encounters of the Bear Kind," what is the author's point of view toward bear researcher, John Hechtel? He takes foolish and unnecessary risks as a researcher.John has deep respect for the black bears he studies.His research proves people can interact safely with bears.His work puts bears' lives at risk. Question 2Part BWhich evidence from the text best supports the answer to Part A?"Most people probably wouldn't want this job.""But John feels that the chance to climb into the private world of a black bear is a great privilege.""'I see a lot of the same traits in bears that I see in dogs, and even peopleincluding curiosity and playfulness.'""John's main worry in his work is not his own safety, but that of the bea Brainliest will be given If a new line, p, is drawn parallel to line I, whichstatement is true?O Line p must be drawn in plane B.O Line p must be perpendicular to line m.O Line p must be drawn so that it can lie in the sameplane as line 1O Line p must be drawn in the same plane as line n. Please help me answer this question can anyone answer this for me? would be greatly appreciated!